数字信号处理习题与答案

更新时间:2023-04-10 20:44:01 阅读量: 实用文档 文档下载

说明:文章内容仅供预览,部分内容可能不全。下载后的文档,内容与下面显示的完全一致。下载之前请确认下面内容是否您想要的,是否完整无缺。

==============================绪论==============================

1. A/D 8bit 5V

00000000 0V

00000001 20mV

00000010 40mV

00011101 29mV

==================第一章 时域离散时间信号与系统================== 1.

①写出图示序列的表达式

答:3)1.5δ(n 2)2δ(n 1)δ(n 2δ(n)1)δ(n x(n)-+---+++=

②用δ(n) 表示y (n )={2,7,19,28,29,15}

2. ①求下列周期

)

54sin(

)8sin()4()51cos()3()5

4sin()2()8

sin(

)1(n n n n n ππ

ππ-

②判断下面的序列是否是周期的; 若是周期的, 确定其周期。

(1)A是常数 8ππn 73Acos x(n)???? ??-= (2))81(j e )(π-=n n x 解: (1) 因为ω=73π, 所以3

14π2=ω, 这是有理数, 因此是周期序列, 周期T =14。

(2) 因为ω=81, 所以ω

π2=16π, 这是无理数, 因此是非周期序列。 ③序列)Acos(nw x(n)0?+=是周期序列的条件是是有理数2π/w 0。

3.加法 乘法

序列{2,3,2,1}与序列{2,3,5,2,1}相加为__{4,6,7,3,1}__,相乘为___{4,9,10,2} 。 移位

翻转:①已知x(n)波形,画出x(-n)的波形图。

② 尺度变换:已知x(n)波形,画出x(2n)及x(n/2)波形图。

卷积和:①h(n)*求x(n),其他0

2n 0n 3,h(n)其他03n 0n/2设x(n) 例、???≤≤-=???≤≤= }2

3,4,7,4,23{0,h(n)*答案:x(n)= ②已知x (n )={1,2,4,3},h (n )={2,3,5}, 求y (n )=x (n )*h (n )

x (m )={1,2,4,3},h (m )={2,3,5},则h (-m )={5,3,2}(Step1:翻转)

解得y (n

)={2,7,19,28,29,15}

③(n)x *(n)x 3),求x(n)u(n u(n)x 2),2δ(n 1)3δ(n δ(n)2、已知x 2121=--=-+-+=

}{1,4,6,5,2答案:x(n)=

4. 如果输入信号为,求下述系统的输出信号。

解:首先写出输入信号的取样值

(a) 该系统叫做恒等系统。

5. ①设某系统用差分方程y (n )=ay (n -1)+x (n )描述,输入x (n )=δ(n )。若初始条件y(-1)=0,求输出序列y (n )。 得x(n)1)ax(n 0及差分方程y(n)1)解:由初始条件y(+-==-

)

()()(,时)2()1()2(,时2)1()0()1(时11

)0()1()0(,时02n u a n y a n y n n a δay y n a

δay ,y n δay y n n n

====+===+===+-==

若初始条件改为y(-1)=1,求y(n)

)()1()(方程,1)1(初始条件n x n ax n y y +-==-

)

()1()()1()(,时)1()2()1()2(,时2)1()1()0()1(,时11)0()1()0(,时02n u a a n y a a n y n n a a δay y n a

a δay y n a

δay y n n n

+=+==+=+==+=+==+=+-==

②设差分方程如下,求输出序列y(n)。0n 0,y(n)δ(n),x(n) , x(n)1)ay(n y(n)>==+-=

))()(()1(解:1n δn y a n y -=--

,)())1()1(()2(,时1))0()0(()1(,时00

))1()1(()0(,时1211

11<-=-=---=--=-=-=-==-==-----n a n y a δy a y n a δy a y n δy a y n n

③设LTI 系统由下面差分方程描述:1)x(n 2

1x(n)1)y(n 21y(n)-++-=。 设系统是因果的, 利用递推法求系统的单位脉冲响应。

解: 令x (n )=δ(n ), 则1)δ(n 2

1δ(n)1)h(n 21h(n)-++-= n=0时,11)δ(2

1δ(0)1)h(21h(0)=-++-= n=1时,12

121δ(0)21δ(1)h(0)21h(1)=+=++= n=2时,2

1h(1)21h(2)== n=3时,221h(2)2

1h(3)???? ??== 所以,δ(n)1)u(n 21h(n)1n +-???

? ??=-

6.离散时间系统

。请用基本组件,以框图的形式表示该系统。

解:

7.① ①判断下列系统是线性还是非线性系统。

解:(a )系统为线性系统。

(b)系统为线性系统。

(c)系统是非线性的。

(d)系统没有通过线性性检验。

?系统没有通过线性性检验的原因并不是因为系统是非线性的(实际上,系统的输入输出表达式是线性的),而是因为有个常数B。因此,输出不仅取决于输入还取决于常数B。所以,当时B≠0,系统不是松驰的,如果B=0,则系统是松驰的,也满足线性检验。

(e)系统是非线性的。

②证明是线性系统。

证:

②证明y(n)=nx(n)系统是移变系统。

证:

③①判断下述系统是因果的还是非因果的。

②下列哪一个单位抽样响应所表示的系统不是因果系统?( D )

A. δ(n)

B. h(n)=u(n)

C. h(n)=u(n)-u(n-1)

D. h(n)=u(n)-u(n+1)

⑤以下序列是LTI 系统的单位序列响应h(n),判断系统的因果性和稳定性。

1)n u(0.34)(2)(1)δ(n n --+

答案 (1)非因果、稳定 (2)非因果、不稳定。

⑥判断题: 一个系统是因果系统的充要条件是,单位序列响应h(n)是因果序列。(错)

8.① 考虑下面特殊的有限时宽序列

。把序列分解成冲激序列加权和的形式。

解: ②将序列x(n)用一组幅度加权和延迟的冲激序列的和来表示 。

∑-=-=

-+-+-+++-=3

1k k)x(k)δ(n 3)x(3)δ(n 2)x(2)δ(n 1)x(1)δ(n x(0)δ(n)1)1)δ(n x(x(n) ③若???≤≤=其他

0402)(n n x n 用单位序列及其移位加权和表示 x(n)= )4(16)3(8)2(4)1(2)(-+-+-+-+n n n n n δδδδδ。

9. ① 一个LTI 系统的单位冲激响应和输入信号分别为

求系统对输入的响

应。

②一个松弛线性时不变系统。求系统对于x(n)的响应

y(n)。

解:用式中的卷积公式来求解

③一个线性时不变系统的冲激响应为。请确定该系统的单位阶跃响应。

解:

④设线性时不变系统的单位脉冲响应h(n)和输入x(n)分别有以下几种情况,分别求输出y(n)。

(1)h(n)=R4(n) , x(n)=R5(n(2)h(n)=2R4(n) , x(n)=δ(n)-δ(n-

解:(1){1,2,3,4,4,3,2,1}

(2){2,2,0,0,-2,-2}

⑤设系统的单位脉冲响应h(n)=u(n),,求对于任意输入序列x(n)的输出y(n),并检验系统的因果性和稳定性

10.①考虑一个LTI,该系统的冲激响应为,确定a的取值范围,使得系统稳定。

解:首先,系统是因果的

因此,系统稳定的条件是|a|<1。否则,系统是不稳定。

实际上,h(n)必须随n 趋于无穷呈指数衰减到0,系统才是稳定的。

②考虑冲激响应为的线性时不变系统,若该系统稳定,则a和b的取值范围为多少?

解:显然系统是非因果的,

所以,系统稳定的条件是 |a|<1 且 |b|>1 。

11.将图示周期矩形脉冲信号展成指数形式傅立叶级数

解:直接代入公式有

12. 数字信号是指___时间幅度都离散的_______的信号。

判断:数字信号处理的主要对象是数字信号,且是采用数值运算的方法达到处理目的的。( 对)

判断:单位阶跃序列与矩形序列的关系是u(n)

N)

u(n

(n)

R

N

-

-

=。(错)

判断:因果系统一定是稳定系统。( 错)

判断:如果系统对输入信号的运算关系在整个运算过程中不随时间变化,则这种系统称为时不变系统。(对)

判断:所谓稳定系统是指有界输入、有界输出的系统。(对)

判断:差分方程本身能确定该系统的因果和稳定性。(错。差分方程本身不能确定该系统的因果和稳定性,还需要用初始条件进行限制。)

判断:若连续信号属带限信号,最高截止频率为Ωc ,如果采样角频率Ωs<2Ωc ,那么让采样信号通过一个增益为T 、 截止频率为Ωs/2的理想低通滤波器,可以唯一地恢复出原连续信号。( 错 。角频率Ωs ≥2Ωc )

设系统的单位抽样响应为h(n),则系统因果的充要条件为( 当n<0时,h(n)=0 )

=======================第二章 z 变换与DTFT =======================

1. ①设x (n )=R N (n ),求x (n )的傅里叶变换。

)2/sin()2/sin(e )e e (e )e e (e e 1e 1e e )()e (解:2/)1(j 2/j 2/j 2/j 2/j 2/j 2/j j j 1

j j j ωωωωωωωωωωωωωωN n R X N N N N N n N n n n N --------∞-∞=-=--=

--=--=

==∑∑ 当N =4时,其幅度与相位随频率ω的变化曲线如图所示:

②序列2)δ(n x(n)-=的傅里叶变换为 ω2j e -。

③设系统的单位脉冲响应h (n )=a n u (n ), 0

完成下面各题: (1) 求出系统输出序列y (n ); (2) 分别求出x (n )、 h (n )和y (n )的傅里叶变换。

2)

u(n 2a u(n)a 2)]δ(n [δ(n)u(n)a x(n)h(n)解:(1)y(n)2n n n -+=-+*=*=- (2)j2ωn j ωn j ω2e 12)]e 2δ(n [δ(n))X(e -∞

-∞

=-+=-+=∑

j ω0n j ωn n n j ωn n j ωae 11

e a u(n)e a )H(e -∞

=-∞

-∞=--===∑∑

j ωj2ω

j ωj ωj ωae 12e 1)X(e )H(e )Y(e ---+=?=

④n))的傅里叶反变换x(。求X(e π |ω|ω0,ω|ω|1,)1、已知X(e jw 0

j ω???≤<<=

πn n sin ωd ωe 2π1解:x(n)0ω

ωj ωn

00==?-

2.

sin(πk/8)

sin(πk/2)

e

)

e

(e

e

)

e

(e

e

e

1

e

1

(n)e

x

(k)

X

解:k8

j

k

8

π

j

k

8

π

j

k

8

π

j

k

2

π

j

k

2

π

j

k

2

π

j

k

4

π

j

jkπ

7

n

kn

8

j

~

~-

-

-

-

-

-

-

=

-

=

-

-

=

-

-

=

=∑

3. ①

4. ①x(n)=u(n), 求其Z变换。

解:

当|z|>1时X(z)存在,因此收敛域为|z|>1

②x(n)=R N(n)的Z变换及其收敛域。(有限长序列)

解:

收敛域为:0<|z|≤∞

③求序列)

(

)

(n

u

a

n

x n

=的Z变换及收敛域。(右边序列之因果序列)

解: n 1211n 0

n 1n n n )(az )(az az 1)(az u(n)z a X(z)---∞=--∞-∞=++++===

∑∑ 这是无穷等比级数,公比是1-=az

q , 在什么情况下收敛?||||即,1||1a z az

><- a z ,a z z az

11所以:X(z)1>-=-=- 本例,极点为z=a 。

④求序列 1)n u(b x(n)n ---=z 变换及收敛域(左边序列之反因果序列)

解:n n 1n 1n 1n n n n z)(b z b 1)z n u(b X(z)∑∑∑∞-∞=--∞=∞

=----=-=---= b z ,b z z z

b 1z b X(z)11<-=--=-- 本例,极点为:z=b

⑤求序列 ???<-≥=0

n b 0n a x(n)n n z 变换及收敛域 解:|b ||z ||a |,b)

a)(z (z b)a z(2z a

z z b z z z a z b X(z)0n n n 1n n n <<----=-+-=+-=

∑∑∞=---∞=- 本例,极点为:z=a,z=b ⑥ u(n)a y(n)n =的Z 变换为 1/(1-az -1) ____ ,收敛域为___∣z ∣>∣a ___。

1)n u(a y(n)n ---=的Z 变换为 1/(1-az -1) ____ ,收敛域为___∣z ∣<∣a ___。

5.①已知X (z )=(1-az -1)-

1,|z |>a , 求其逆Z 变换x (n )。(留数法) 解:

n ≥0时,F (z )在c 内只有1个极点:z 1=a ; n <0时,F (z )在c 内有2个极点:z 1=a , z 2=0(高阶);

②PPT例11(留数法)

③PPT例12(部分分式展开法)

④(考原题!!!!!!!!!!)已知4

z

,

)

4

1

z)(z

(4

z

X(z)

2

>

-

-

=,求z反变换。

解:

是因果序列。

)

(

是右边序列,故

)

(

处,

的收敛域包含

)

(

,即

1

)

(

lim

n

x

n

x

z

X

z

X

z

-

=

所以当n<0时,x(n)=0。只需考虑n≥0时的情况。

=

?

=-1

)

(

)

(n z

z

X

z

F

)

4

1

z)(z

(4

z1

n

-

-

+

如图所示,取收敛域的一个围线c,可知

当n≥0时, C内有两个一阶极点4

,

4

/

1=

=z

z,

所以

[]0

,

4

4

15

1

)]

4

1

)(

4

/(

[

Res

)]

4

1

)(

4

/(

[

Res

)

(

2

4

1

1

4

1

-

=

-

-

+

-

-

=

+

-

=

+

=

+

n

z

z

z

z

z

z

n

x

n

n

z

n

z

n

[]

??

?

?

?

<

-

=

+

-

,

4

4

15

1

)

(

2

n

n

n

x

n

n

⑤已知4

z

4

1

,

)

4

1

z)(z

(4

z

X(z)

2

<

<

-

-

=,求z反变换。

=?=-1)()(解:n z z X z F )41)(4(1

--+z z z n

如图所示,取收敛域的一个围线c ,

分两种情况讨论:

(1)n≥-1时,C 内只有一个一阶极点z=1/4

1,41514/14)4/1()]41)(4/()41([)]41)(4/([Res )(14

11411-≥?=-=---=-

-=-+=+=+n z z z z z z z n x n n z n z n )1(415

1)(或记作:+?=-n u n x n (2)当n<-1时,

C 内有极点:z=1/4(一阶), z=0(高阶);

而在C 外仅有 z=4(一阶)这个极点,且F (z)的分母多项式比分子多项式的最高次数高2阶以上,

1n ,41511/444)]41z)(z /(4Res[z x(n)2n 1n 4z 1

n -

1)(或记作:2--?=+n u n x n ???????-≤-≥=+-2,415

11,4151)(因此2n n n x n n )2(15

4)1(154)(或记作:2--++=+-n u n u n x n n 6.①已知

,分析其因果性和稳定性。 解 H (z )的极点为z =a , z =a -1。

(1) 收敛域为a -1<|z |≤∞: 对应的系统是因果系统,但由于收敛域不包含单位圆,因此是不稳定系统。

(2) 收敛域为0≤|z |

(3) 收敛域为a<|z|

②时域离散线性时不变系统的系统函数为b为常数,a,b)a)(z (z 1H(z)--=

。若要求系统稳定,则a 的取值域为__|a |≠1__和 b 的取值域为____|b |≠1____。

③时域离散线性时不变系统的系统函数为b为常数,a,b)

a)(z (z 1H(z)--=。若要求系统因果稳定,则a 的

取值域为__0≤|a |<1__和 b 的取值域为___0≤|b |<1__。

④8、如果系统函数用下式表示: )

0.9z )(10.5z (11H(z)11----=。则下列关于收敛域的说法正确的是( D ) A .该系统无法通过选择适当的收敛域使该系统因果稳定

B .收敛域为|z|<0.5 时,系统因果稳定

C .收敛域为0.5<|z|<0.9时,系统因果稳定

D .收敛域为|z|>0.9时,系统因果稳定

7.①已知系统的差分方程为:1b 0,x(n)1)by(n y(n)<<+-=。指出系统函数的零极点并分析系统的频响特性。

解:系统的传输函数为: b |z |b z z bz 11H(z)1>-=-=

-

∴极点为z=b ,零点为z=0

②已知H (z )=1-z -N ,试定性画出系统的幅频特性。 解

极点:H(z)的极点为z=0,这是一个N 阶极点,它不影响系统的幅频响应。

零点:零点有N 个,由分子多项式的根决定。

③已知某数字滤波器的系统函数为:1

0.9z 11H(z)--= (1)画出零极点分布图

(2)利用几何确定法分析幅度特性,画出幅度特性图;

(3)试判断滤波器的类型(低通、 高通、 带通、 带阻)。

解: (1)将系统函数写成下式: 0.9z z =0.9z 11H(z)1

--=- 系统的零点为z =0, 极点为z =0.9,零点在z 平面的原点,零极点分布图为:

(2)不影响频率特性,而惟一的极点在实轴的0.9处,幅度特性图为:

(3)滤波器的通带中心在ω=0处,这是一个低通滤波器。

8.下列关系正确的为(D )

判断:时域离散信号傅里叶变换存在的充分条件是序列绝对可和。(对)

判断:序列的傅里叶变换是频率ω非周期函数。(错。序列的傅里叶变换是频率ω的周期函数,周期是2π。)判断:序列z变换的收敛域内可以含有极点。( 错)

若H(Z)的收敛域包括∞点,则h(n)一定是__因果_序列。

线性时不变系统h(n)是因果系统的充要条件是________ h(n)=0,n<0 或收敛域在某圆的外面____________。

线性时不变系统h(n)是稳定系统的充要条件是_________ h(n)绝对可和或收敛域包括单位圆___________。

序列的傅里叶变换等于序列在( 单位圆)上的Z变换。

====================第三章离散傅里叶变换(DFT)=================== 1.①已知)

(

)

(

4

n

R

n

x=,分别求8和16点DFT

解:∑

=

-

=

-

-

=

=

=

=

=

3

n

kn

j

7

n

8

2πkn

j

4

nk

N

1

N

n

8

e

(n)e

R

W

x(n)

X(k)

8时

(1)N

7

k

sin(πk/8)

sin(πk/2)

e

e

1

e

1k

j

k

j

4k

j

8

8

8

=

-

-

=-

-

-

=

-

=

-

-

=

=

=

=

=

3

n

kn

j

15

n

16

2πkn

j

4

nk

N

1

N

n

16

e

(n)e

R

W

x(n)

X(k)

16时

(2)N

15k 0sin(πk/16)sin(πk/4)

e e 1e 1k j k j 4k j 163π162

π16

2

π≤≤=--=---

频率采样点数不同,DFT 的长度不同,DFT 的结果也不同。

③假设线性时不变系统的单位脉冲响应h(n)和输入信号x(n)分别用下式表示(n)R x(n)(n),

R h(n)48==

(1)计算该系统的输出信号y(n) (2)如果对x(n)和h(n)分别进行12点DFT ,得到X(k)和H(k),令X(k)H(k)(k)Y 1?= (k)]IDFT[Y (n)y 1

1= 求y1(n)

解:(1)y(n)={1,2,3,4,4,4,4,4,3,2,1}

(2)y1(n)={ 1,2,3,4,4,4,4,4,3,2,1,0}

2. ①计算下面给出的两个长度为4的序列h (n )与x (n )的4点和8点循环卷积。

解:h (n )与x (n )的4点循环卷积矩阵形式为

h (n )与x (n )的8点循环卷积矩阵形式为

③已知长度为N=10的两个有限长序列:

(1)做图表示x1(n)、x2(n)

(2)求y1(n)=x1(n) * x2(n)

(3)求y2(n)=x1(n) 圈* x2(n),循环卷积区间长度L=10。

3.利用DFT的共轭对称性,设计一种高效算法,通过计算一个N点DFT,就可以计算出两个实序列x1(n)和x2(n)的N点DFT。

解:构造新序列x(n)=x1(n)+jx2(n),对x(n)进行DFT,得到:

4. 对6点有限长序列{5,1,3,0,5,2}进行向左2点圆周移位后得到序列__{3,0,5,2,5,1} __。

5.已知y(n)=x(n)*h(n),x(n)和h(n) 的长度分别为M和N。x(n)和h(n)的L点循环卷积(L>M,L>N)用w(n)表示,w(n)=y(n)的条件是___L≥M+N-1___________。(循环卷积等于线性卷积的条件)

离散傅里叶变换中,有限长序列都是作为周期序列的一个周期来表示的,都隐含有周期性的意思。( 对)

====================第四章快速傅里叶变换(FFT)=================== 1.画出16点基2DIT-FFT和基2DIF-FFT的运算流图,并计算其复乘和复加的计算量。

2.一个蝶形运算,需要_____一___次复数乘法和___两_____次复数加法运算。

对于N点(N=2M)的按时间抽取的基2FFT算法,共需要作MN/2 次复数乘和_ MN___次复数加。

下列关于FFT说法错误的是( B )。

A. DIF-FFT算法与DIT-FFT算法的运算量一样。

B. DIT-FFT算法输入序列为自然顺序,而输出为倒序排列。

C. DIF-FFT算法与DIT-FFT算法的蝶形运算略有不同,DIF-FFT蝶形先加(减)后相乘,而DIT-FFT蝶形先乘后加(减)。

D. FFT算法就是不断地把长序列的DFT分解成几个短序列的DFT来减少DFT的运算次数。

循环卷积与数字卷积的关系(只记结论)

===================第五章数字滤波器的基本结构=================== 1.

2.①

④已知一个IIR滤波器的系统函数为

2

13z

2z

1

1

H(z)

-

-+

-

=

则此滤波器的直接型结构表示为_。

⑤假设滤波器的单位脉冲响应为1

a

u(n)

a

h(n)n<

<

=。求出滤波器的系统函数,并画出它的直接型结构。解:a

z

az

1

1

ZT[h(n)]

H(z)

1

>

-

=

=

-

⑥已知系统的单位脉冲响应为:

5)

0.5δ(n

3)

2.5δ(n

2)

0.3δ(n

1)

2δ(n

δ(n)

h(n)-

+

-

+

-

+

-

+

=

试写出系统的系统函数,并画出它的直接型结构。

解:将进行Z变换,得到它的系统函数5

3

2

10.5z

2.5z

0.3z

2z

1

H(z)-

-

-

-+

+

+

+

=

3.

4. ①若数字滤波器的结构如图所示:则它的差分方程为 y (n )=2y (n -1)-0.8y (n -2)+x (n )+3x (n -1) , 系统函

数为211

0.8z

2z 13z 1H(z)---+-+=。

② 图中画出了四个系统, 试用各子系统的单位脉冲响应分别表示各总系统的单位脉冲响应, 并求其总系统函数。 解:(1) h (n )=h 1(n )*h 2(n )*h 3(n ), H (z )=H 1(z )H 2(z )H 3(z

(2) h (n )=h 1(n )+h 2(n )+h 3(n ), H (z )=H 1(z )+H 2(z )+H 3(z

(3) h (n )=h 1(n )*h 2(n )+h 3(n ), H (z )=H 1(z ) H 2(z )+H 3(z

(4) h (n )=h 1(n )*[h 2(n )+h 3(n )*h 4(n )]+h 5(n )

=h 1(n )*h 2(n )+h 1(n )*h 3(n )*h 4(n )+h 5(n ),

H (z )=H 1(z )H 2(z )+H 1(z )H 3(z )H 4(z )+H 5(z )

本文来源:https://www.bwwdw.com/article/4zrl.html

Top